Difference between revisions of "2018 AMC 10A Problems/Problem 1"

m
m (Solution)
Line 7: Line 7:
 
== Solution ==
 
== Solution ==
  
Working it out gives you 11/7
+
Working it out gives you <math>\boxed{\textbf{(B) } 11/7}</math>.
  
 
== Video Solutions ==
 
== Video Solutions ==

Revision as of 12:47, 2 February 2021

Problem

What is the value of \[\left(\left((2+1)^{-1}+1\right)^{-1}+1\right)^{-1}+1?\]

$\textbf{(A) } \frac58 \qquad \textbf{(B) }\frac{11}7 \qquad \textbf{(C) } \frac85 \qquad \textbf{(D) } \frac{18}{11} \qquad \textbf{(E) } \frac{15}8$

Solution

Working it out gives you $\boxed{\textbf{(B) } 11/7}$.

Video Solutions

https://youtu.be/vO-ELYmgRI8

https://youtu.be/cat3yTIpX4k

~savannahsolver

https://youtu.be/19mpsCcQzY0

~Education, the Study of Everything

See Also

2018 AMC 10A (ProblemsAnswer KeyResources)
Preceded by
First Problem
Followed by
Problem 2
1 2 3 4 5 6 7 8 9 10 11 12 13 14 15 16 17 18 19 20 21 22 23 24 25
All AMC 10 Problems and Solutions

The problems on this page are copyrighted by the Mathematical Association of America's American Mathematics Competitions. AMC logo.png